Homework Statement
A particle has the potential energy shown in the figure. (Figure 1) What is the
x-component of the force on the particle at x =5, 15, 25, and 35 cm?
I'm stuck on finding the Fx at 25 & 35 cm.Homework Equations
F = -du/dx
100 N = 1J/cm
The Attempt at a Solution
Slope...
Homework Statement
Here are the problem statement and the solution. I'm stuck at where the book suggests the formulas for the x and y coordinations (highlighted in yellow) of the third charge. Any explanations or proof on how they came to the conclusion for the third charge coordinations would...
I am trying to change from J2000 to Apparent positions. Astronomical Algorithms by Jean Meeus addresses but does not give a correct answer. VSOP87 gives the Apparent or Of Date positions in VSOP87C and VSOP87D but I need to calculate directly from J2000. How do I do this?
Homework Statement
A boat leaves the dock at t=0 s and heads out into the lake with an acceleration of 2m/s^2i.A strong wind pushing the boat, giving it additional velocityof 2m/si+ 1m/sj.
(a) What is the velocity of the boat at t=10s?
(b) What is the position of the boat at t=10?
Homework...
How would ##[p_x, r]## be expanded? Where ##r=(x,y,z)##, the position operators. Do you do the commutators of ##p_x## with ##x, y,z## individually? So ##[p_x,x]+[p_x,y]+[p_x,z]## for example?
Hey,
I'm looking for a bit of advice with regards to applying for a PhD program in Europe.
As a bit of background: I graduated from Durham University in 2013 with a 1st class MPhys in Theoretical Physics. My thesis (which contributed to half of the grading in the 4th, masters year) was focused...
Homework Statement
From the diagram below find the position of the man (##x##) if the system is in balance. Total length is ##L## and the man is distance ##x## from one end.
Homework Equations
3. The Attempt at a Solution [/B]
I know that the system must be in balance if all the torque and all...
Hello Forum,
My understanding is that the state of the system is ##|\Psi>##. We can take the inner product between the state ##|\Psi>## and the eigenstates of the position operator ##\hat{x}##:
$$<x|\Psi>=\Psi(x)$$
The function ##\Psi(x)## is the wave function we are initially introduced to in...
Homework Statement
So, I'm doing this problem from Townsend's QM book
6.2[/B]
Show that <p|\hat{x}|\psi> = i\hbar
\frac{\partial}{\partial p}<p|\psi>
Homework Equations
|\psi(p)> = \int_\infty^{-\infty} dp |p><p|\psi>
The Attempt at a Solution
So,
<p|\hat{x}|\psi>
= <p|\hat{x}...
I'm trying to make a simple HTML page that has a few elements. The problem is that when I zoom in using the mouse wheel and holding down Ctrl everything starts to go all over the place and it just looks completely mixed.
I tried to find solutions to this but have had no luck so far.
I want my...
I understand that the Fourier transform to obtain the representation of a wavefunction in k space is
$$ \phi(k) =\frac{1}{2\pi}\int{dx \psi(x)e^{-ikx} } $$
and that $$p=\bar{h} k$$
But why then is $$\phi(p) =\frac{\phi(k)}{\sqrt{\bar{h}}} $$
Many thanks in advance :)
First term of the progression is 3 & the common difference is 4
Find the sum of the first 20 terms of the progression that is obtained by removing the terms in the even positions of the given progressions, such as the second term,fourh term, sixth term.
Formula preferences
For the sum of an...
Homework Statement
http://imgur.com/a/FDfAp
What is the phase constant?
Homework Equations
x(t) = A*cos(ωt+Φ)
The Attempt at a Solution
If I'm not mistaken at t = 0 the graph starts at half the amplitude or 5. Also the amplitude of this graph is 10, and at t = 0 angular velocity is also 0.
5...
How much force is required to turn the front wheel of a 4 wheel robot that weighs 200lb. Assume the wheel is 6inches wide, has a diameter of 12 inches (rubber). I would like to know how much force is required to turn a single wheel left or right when it is in stationary (non-moving) position...
Homework Statement
An object is moving in a straight line with a constant acceleration. Its position is measured at three different times, as shown in the table below.
Time (s) | Position, (m)
48.40 | 9.100
49.80 | 17.479
51.20 | 32.816
Calculate the magnitude of the acceleration at t=49.80 s...
Homework Statement
Bead on spoke:
constant speed ##u## along spoke
it starts at center at ##t=0##
angular position is given by ##\theta=\omega t##, where ##\omega## is a constant
Homework Equations
## \frac{d\hat r}{dt} = \dot \theta \hat \theta ## (1)
## \frac{d\hat \theta}{dt} = -\dot...
Homework Statement
Find the amount of work done between points x1 and x2.
Force at x1: F=ax1
Force at x2: F=ax2
Homework Equations
F=ma
W=F (dot product) S
The Attempt at a Solution
W=ax2*(x2-x1)
Hi!
I don't know if I'm in the right forum of this site but I'm trying anyway. I was just wondering if someone could explain how the step- and impulse response is related to an angular position (of e.g. a spacecraft )? Just a little about the theory since I am trying to actually understand...
Homework Statement
What is the final position of the object if its initial position is x = 0.40 m and the work done on it is equal to -0.19 J?
Homework Equations
work(J)= Fx ΔX
The Attempt at a Solution
I do not know where to begin? please give me a hint where to begin.
I am wondering about the minimum possible uncertainty (standard deviation) in an electron's position (Δx). How precise can one know the electron's whereabouts without creating other sorts of particles and phenomenons.
I know of the localization energy interpretation of the energy uncertainty...
Homework Statement
The position of an object that is oscillating on an ideal spring is given by the equation x =
(12.3 cm) cos[(1.26s-1)t]. At time t = 0.815 s,
(a) how fast is the object moving?
(b) what is the magnitude of the acceleration of the object?
Homework Equations
As follow
The...
$\tiny{205.o27.12 }$
$\text{Given the velocity $v=\frac{ds}{dt}$}$
$\text{and the initial position of a body moving along a
coordinate line,}$
$\text{find the body's position at time t}$
\begin{align}
\displaystyle
v&=\sin\left({\pi t}\right) &s(0)&=20 \\
v'&=\cos{(\pi t)} \\
s(t)&=...
Homework Statement
Prove that ##[L_i,x_j]=i\hbar \epsilon_{ijk}x_k \quad (i, j, k = 1, 2, 3)## where ##L_1=L_x##, ##L_2=L_y## and ##L_3=L_z## and ##x_1=x##, ##x_2=y## and ##x_3=z##.
Homework Equations
There aren't any given except those in the problem, however I assume we use...
i am having a hard time understanding why do we place the brushes on the Magnetic neutral axis, the textbook simply says, because this is where the current reversal takes place, is the point where emf from different meet is the same as the point of current reversal?
Homework Statement
The figure shows a two-ended “rocket” that is initially stationary on a frictionless floor, with its center at the origin of an x axis. The rocket consists of a central block C (of mass M = 6.40 kg) and blocks L and R (each of massm = 1.90 kg) on the left and right sides...
I've made a drawing in order to visualize the problem better:
1. Homework Statement
A car can increase its speed only at 3.00 m/s2, move at constant velocity, or decrease its speed at 7.00 m/s2. Starting at rest, the driver wishes to drive to a road sign located at x=100 m. He increases his...
What are the chances of getting an Entry-Level Electrical Engineering job 3 years after I graduated? I had a 3 year gap because I was getting my teaching degree. However, I did stay current with electrical engineering fundamentals / knowledge during those years.
Homework Statement
An object of 5 kg is acted on by a force and moved from 0.5 to 2.0m.
How much work does the force do?
As the object moved, friction with a coefficient of 0.2 also acted upon it. What is its final speed if it started at rest?
Homework EquationsThe Attempt at a Solution
So...
Homework Statement
A force of 10 Newtons can stretch a spring by 0.04 m. Suppose a mass of 5 kg is attached to the lower end of the spring. We stretch the mass downward by 0.05 m from its equilibrium position and release it from rest. Determine the position of the mass relative to its...
Homework Statement
My homework problem is a proof in orbital mechanics, but I'm not looking for specific help on that just yet, I'd like to work through it on my own. In doing so however, I'm having a hard time conceptualizing the idea of derivatives of vectors at a specified time. If r is a...
Hello,
My question is probably geared towards the professors or postdocs reading this forum and concerns the recruitment aspect of the professors in the US.
I am a European physics student currently visiting a "top" Californian university. During this short visit (8 months) I have come to...
What is the relation between the correlators ##\langle 0 | T\phi(x_{1})\phi(x_{2}) | 0 \rangle## and ##\langle 0 | T\phi(p)\phi(x=0) | 0 \rangle##?
I can derive the momentum space Feynman rules for ##\langle 0 | T\phi(x_{1})\phi(x_{2}) | 0 \rangle##. Are the momentum space Feynman rules for...
A particle's position is given by $$r_i=r_i(q_1,q_2,...,q_n,t)$$ So velocity: $$v_i=\frac{dr_i}{dt} = \sum_k \frac{\partial r_i}{\partial q_k}\dot q_k + \frac{\partial r_i}{\partial t} $$
In my book it's given $$\frac{\partial v_i}{\partial \dot q_k} = \frac{\partial r_i}{\partial q_k}$$...
Homework Statement
Compute ##\displaystyle{\int\ \frac{d^{4}p}{(2\pi)^{4}}} \frac{i}{p^{2}-m^{2}+i\epsilon}e^{-ip \cdot{(x-y)}}## in terms of the invariant interval.
Interpret your answer in the limit of small and large invariant intervals and for zero mass.
Homework Equations
The Attempt...
I could completely understand the fact it it was just a limit to our observations but how can it be a property of the microscopic particle itself? Here's how I understand about probabilities:
Before a die is thrown, the probability of a certain number coming up is 1/6. But, it's before the die...
I have a problem where I need to figure out the initial velocity vector \vec{v_0} of a projectile, in order for it to land at the final position \vec{r_f} = x_f\hat{x} + y_f\hat{y} + z_f\hat{z}, from initial position \vec{r_0}.
___
The only knowns in the problem are \vec{r_0} and \vec{r_f}...
Homework Statement
if displacement is the shortest distance between two point then that would also meant that it is equivalent to the hypotenuse when using phytogaras theorem right?. but why isn't displacement the shortest length between two point ?because Xinitial - Xfinal couldn't posibly be...
Homework Statement
Finding yourself in a nearly empty airplane, you take a seat other than your allotted seat. Soon, an air hostess comes up to you and requests you to occupy a different seat.
What may be the reason?
A) The plane momentum can change if the passengers change their position...
Homework Statement
FInd the angle theta between AB and AC. a-1.7m, b=1.3m
masteringengineering.com says my answer is wrong. I'm not sure of any other way to do the problem. Please advise.
Homework Equations
A(3, 0, 0) B(0, -.75, 1.3) C(0, 1.7, 1.5)
The Attempt at a Solution
r(AC)=...
Hi!
I hope I'm posting in the correct section. I found this very similar topic in this forum: https://www.physicsforums.com/threads/orbital-ellipse.482713/
Aside from very helpful formula for calculating semi-major axis, the question was left unanswered.
I have a working simulation of...
Homework Statement
The position function of a projectile is given by r(t) = (5t + 6t2) m I + (30 - t3) m j. What is the displacement of the particle in magnitude angle form at the = 2.0 seconds.
Homework Equations
√x2+y2
tan-1 = y/x
Possibly: x = v0t + 1/2at2The Attempt at a Solution
[/B]
I...
hi, initially I want to put into words that I looked up the link (http://physics.stackexchange.com/questions/86824/how-to-get-the-position-operator-in-the-momentum-representation-from-knowing-the), and I saw that $$\langle p|[\hat x,\hat p]|\psi \rangle = \langle p|\hat x\hat p|\psi \rangle -...
In most situations in QM we would get a quantized energy basis, that is a countably infinite basis ( I think it's called having a cardinality of aleph 0), In the meanwhile we take the position basis to be continuous ( cardinality of aleph 1?) and I'm pretty sure that there is a theorem stating...
Say a particle was measured to be at a point C. Immediately after this, if i make another measurement, its given in griffith's book that the particle will still be found at C. Isn't this only possible if the particle was at rest?
Homework Statement
Two particles 1 and 2, each carrying 6.0 nC of charge, are located along an x axis, one particle atx=−30 mm and the other at x = 30 mm. Where along the y-axis is a particle 3 carrying a charge of +2.0 nC if it experiences an electric force of 6.9 × 10^−5 N ȷ^?
Homework...
In a closed water heating system with a heat exchanger as the primary heat source (district heating), should the circulator ideally be installed before or after the heat exchanger? Is one or the other better for the exchanger itself?
My textbook (on celestial mechanics) makes a passing reference to position on an ellipse being expressed as:
##r = a(1 - e \cos E)## before moving on to the substance of the chapter. E is the eccentric anomaly, and r is the distance from the focus to the point on the ellipse.
I'm trying to...
Well i am noobie to quantum physics so i matbe totally incorrect so please bear with me.
I had question how is position operator defined mathematically.
I was reading the momentum position commutator from...